You are on page 1of 70

1

CHAPTER 1 SIMPLE STRESS

1-a. Introduction
Mechanics of materials, or strength of materials, is a branch of applied mechanics that deals with the behavior of
solid bodies subjected to various types of loading. It deals with the relations between externally applied loads and their
internal effects. Here, we examine the stresses and strains that occur inside real bodies that deform under loads. Strength
of materials is limited to the study of deformable bodies at rest.
The purpose of studying strength of materials is to ensure that the structures used will be safe against the
maximum internal effects that may be produced by any combination of loading. The engineer must consider both
dimensions and material properties to satisfy requirements of strength and rigidity.

1-b. Internal Forces

We consider a body acted upon by a balanced force system as shown in Fig. 1-1a. To be able to investigate the
internal distribution of forces, we pass an exploratory section a-a through the body thereby exposing the internal forces
acting on the exploratory section that are necessary to maintain equilibrium of either segment. Internal forces and couples
act on the cut section as illustrated in Fig. 1-1b and have magnitudes necessary to produce equilibrium for the free-body.
Specifically, we classify the forces and couples as in the figure. y

a
F1 F1
Mx
F4
Pxy
Pxz
Pxx x
Mxx
F2 F2
F3 Mxz
a z
(a) (b)
Fig. 1-1

The force vector acting along the axis of the body, Fx, represents either a tensile force which tends to elongate the
member or a compressive force that tends to shorten it. This component measures the pulling or pushing action
perpendicular to the section. It is called axial force and often denoted by F.
The remaining two force vectors called shear forces, Fx and Fx, produce shear loading on the cut section. These
are components of the total resistance to sliding the portion to one side of the exploratory section past the other. It is
usually designated as V.
The axial couple vector, Mx, represents a twisting couple whose direction is determined using the right-hand-
screw rule. This twisting couple, referred to as torque, causes a shear action to occur on the cut cross section. It measures
the resistance to twisting the member and is commonly denoted by T.
The other two couples, My and Mz, are referred to as bending moments which are denoted by M. These
components measure the resistance to bending the member about the y and z axes.

1-c. Simple Stress


The fundamental concepts of stress can be illustrated by considering a prismatic bar that is loaded by axial forces
P at the ends, as shown in Fig. 1-2a. A prismatic bar is a straight structural member having a constant cross section
throughout its length. The internal stresses produced in the bar by the axial forces are exposed if an imaginary cut at a
section, say a-a, is made. Because this section is taken perpendicular to the longitudinal axis of the bar, it is called a cross-
section.
a a

P P P

(a) a (b) a
Fig. 1-2
2

Fig. 1-2b shows the isolated part of the bar to the left of section a-a. The tensile load P acts at the left end of the
free-body and at the other end, the forces representing the action of the removed part of the bar upon the part that remains.
These forces are continuously distributed over the cross section. The intensity of the force is called stress. Stress, which is
usually denoted as , is defined as force per unit area, i. e.,

P Equation 1.1
A
where: = stress
P = applied load
A = cross-sectional area
Dividing load by the area does not give the stress at all points in the cross-sectional area; it merely gives the
average stress. The condition under which the stress is constant or uniform is known as simple stress. Uniform stress
distribution can exist only if the resultant of the applied loads passes through the centroid of the cross-section. As a proof,
consider the free-body diagram shown in Fig. 1-3. The resisting forces over the cut section must balance the applied load
P. A typical resisting force is dP. Applying the force and moment conditions of equilibrium, we obtain
y
dP
P Pb
x Pb =
b C
Pb =
x Pb =
P=A

P
z
Fig. 1-3
Hence, the coordinate b of the point C is recognized as being the x-coordinate of the centroid of the section. We,
therefore, conclude that a uniform stress distribution is obtained only when the resultant of the applied loads passes
through the centroid of that surface.

1-c.1. Normal Stress/Axial Stress


When the forces acting at the ends of a bar are directed along the axis of the bar, it is said to be under axial load
(axially loaded). If the bar is stretched by the forces P as shown in Fig. 1-2a, the resulting stresses are tensile stresses; if
the forces cause the bar to be compressed, the stresses are compressive stresses. Since these stresses act in a direction
perpendicular to the cut surface, they are referred to as normal stresses. Equation 1.1 gives the normal stress in a member
under axial loading.
However, if a two-force member is loaded axially but eccentrically as shown
in Fig. 1-4a, we find the conditions of equilibrium of the portion of member
shown in Fig. 1-4b that the internal forces in a given section must be
equivalent to a force P applied at the centroid of the section and a couple M of
magnitude M = Pd. The distribution of stresses cannot be uniform.

(a) (b)

Fig. 1-4
3

1-c.2. Shearing Stress


Shearing stress is caused by a force acting along or parallel to the area resisting the forces and is frequently called
a tangential stress. This stress is produced whenever the applied loads cause one section of the body to tend to slide past
its adjacent section. The average shearing stress across a section can be computed from the formula

V Equation 1.2
A
where: = shearing stress
V = shearing force
A = cross-sectional area

Some examples of shearing stress are single shear (Fig. 1-5a) when, in a riveted connection, a rivet resists shear
across its cross-sectional area while if the rivet resists shear across two cross-sectional areas, it is called double shear (Fig.
1-5b). When a hole is to be punched out of a plate, the plate is subjected to a punching shear (Fig. 1-5c). If shear occurs
over an area parallel to the applied load, it is called direct shear in contrast to the induced shear that may occur over
sections inclined with the resultant load (Fig. 1-5d)

(a) (b)
F1
a
P

R
F2
a
(c) (d)
Fig. 1-5

1-c.3. Bearing Stress


Bearing stress differs from compressive stress in that the Equation 1.3
latter is the internal stress caused by a compressive force whereas
the former is a contact pressure between separate bodies. Some where: = bearing stress
examples of bearing stress are the soil pressure beneath piers and P = compressive load
the forces on bearing plates. A = cross-sectional area
Bolts, rivets, and pins create stresses on the points of contact or bearing surfaces of the members they connect.
The resultant of the force distribution on the surface is equal and opposite to the force exerted on the pin. Corresponding
average force intensity is called the bearing stress.

Fig. 1-6
4

EXERCISES

1. The cross section of a concrete corner column that is loaded uniformly in compression is shown in the figure. (a)
Determine the average compressive stress in the concrete if the load is equal to 3200 k. (b) Determine the coordinates
xc and yc of the point where the resultant load must act in order to produce uniform normal stress in the column. (Gere,
2009) Ans. a) 2.13 ksi; b) (19.22 in, 19.22 in)

2. A machine element is connected at the left end and is subjected to axial loads as shown. Compute the axial stress in
the bar. The area of the bar is 1.125 10-3 m2. (Pytel, 1987) Ans. 120 MPa

225 kN 45 kN 90 kN

A B C D

3. An aluminum rod is rigidly attached between a steel rod and a bronze rod as shown in the figure. Axial loads are
applied at the positions indicated. Find the maximum value of P that will not exceed a stress in steel of 140 MPa, in
aluminum of 90 MPa, or in bronze of 100 MPa. (Pytel, 1987) Ans. 10,000 N

4P P 2P

Steel Aluminum Bronze


A = 500 mm2 A = 400 mm2 A = 200 mm2

4. The truss in the figure is loaded as shown. The area of all truss members is 2500 mm2. Compute the axial stress in
each member of the truss. (Pytel, 1987) Ans. AC = 679.247 MPa; BC = 576 MPa; CD = 120 MPa; AD = AB = 0
300 kN
A
D

2.5 m

C
B 4m
600 kN

5. Determine the largest weight W that can be supported by the two wires shown in the figure. The stress in either wire is
not to exceed 30 ksi. The cross-sectional areas of wires AB and AC are 0.4 in2 and 0.5 in2, respectively. (Pytel, 1987)
Ans. 17.065 kips

B C

30 50
A

W
5

6. In the figure, assume that two- bolts are used to join the three plates. Given that the axial force is 50,000 lb and the
normal stress is not to exceed 18,000 psi, compute the width of each plate to satisfy the normal stress requirements.
Ans. wA = 15.34 in; wB = 7.85 in; wC = 7.06 in
t1 = 0.1 in

B t3 = in

t2 = 0.25 in

7. Determine the cross-sectional areas of members AG, BC and CE for the truss shown. The stresses are not to exceed 20
ksi in tension or 14 ksi in compression. A reduced stress in compression is specified to reduce the danger of buckling.
(Pytel, 1987) Ans. AAG = 1.167 in2; ABC = 5.151 in2; ACE = 1.429 in2
D

6 ft

C 4 ft
E

6 ft

4 ft
B F

6 ft
25 kips
A 4 ft
G

40 kips
8. An aluminum bar AB is attached to its support at A by a pin of diameter 16 mm. The bar has a thickness of 15 mm and
a width of 40 mm. If the allowable tensile stress in the bar is 150 MPa and the allowable shear stress in the pin is 85
MPa, find the allowable load P. Ans. P = 34.181 kN

9. The members of the structure in the figure weigh 200 lb/ft. Determine the smallest diameter pin that can be used at A
if the shearing stress is limited to 5000 psi. Assume single shear. (Pytel, 1987) Ans. 0.52 in
6

10. Compute the shearing stress in the pin at B for the member supported as shown in the figure. The pin diameter is 20
mm. (Pytel, 1987) Ans. 94.022 MPa

40 kN
A 35

200 mm

B
C
250 mm

11. The frame shown in the figure is pin-connected at all joints. Neglect the weight of the members and assume all pins
7
are in. in diameter. Compute the following: (Buchanan, 1988)
16
a) The average shear stress in the pin at A. Ans. 186 ksi
b) The bearing stress between the pin and the support at A. Ans. 170.667 ksi
c) The area of member AB for an allowable normal stress of 16,000 psi. Ans. 3.5 in2

12. The lap joint shown is fastened by four -in diameter rivets. Calculate the maximum safe load P that can be applied if
the shearing stress in the rivets is limited to 14 ksi and the bearing stress in the plates is limited to 18 ksi. Assume the
applied load is uniformly distributed among the four rivets.

P P
4 in

P in
P
in
7

13. Each of the four vertical links has an 8 36 mm uniform rectangular section and each of the four pins has a diameter
of 16 mm. Determine the maximum value of the average normal stress in the links connecting points B and D and
points C and E. (Beer, 2012) Ans. BD = 101.6 MPa; CE = 21.7 MPa

14. The inclined member in Fig. 2 is subjected to a compressive force of 600 lb. Determine the average compressive
stress along the areas of contact defined by AB and BC, and the average shear stress along the horizontal plane defined
by EDB.

A C
E B
D

15. The figure shows a roof truss and the detail of the riveted connection at joint B. Using allowable stresses of 70 MPa
for shear and 140 MPa for bearing, how many 19-mm diameter rivets are required to fasten member BC to the gusset
plate? What is the largest average axial stress in member BC? (Pytel, 1987) Ans. 7 rivets; 128 MPa
D

B F 14 mm gusset
6m
plate

A H
4m C 4m E 4m G 4m 75 75 13 mm

96 kN 200 kN 96 kN PBE
75 75 6 mm

PBC
8

1-d. Stress on an Oblique Plane Under Axial Loading


Consider the two-force member of Fig. 1-7a which is subjected to axial forces P and P'. If we pass a section
forming an angle with a normal plane and draw the free-body diagram of the portion of member located to the left of
that section, Fig. 1-7b, we find from the equilibrium conditions of the free body that the distributed forces acting on the
section must be equivalent to the force P.

(a)

= =

(b) (c) (d)


Fig. 1-7

Resolving P into components F and V, respectively normal and tangential to the section (Fig. 1-7c), we have
F = P cos V = P sin Eq. 1.4
The force F represents the resultant of normal forces distributed over the section, and the force V the resultant of shearing
forces (Fig. 1-7d). The average values of the corresponding normal and shearing stresses are obtained by dividing,
respectively, F and V by the area A of the section:
F V
Eq. 1.5
A A
Substituting for F and V from Eq. 1.4 and Eq. 1.5, and observing from Fig. 1-7c that Ao = A cos , or A = Ao/cos , where
Ao denotes the area of a section perpendicular to the axis of the member, we obtain
P P
cos2 and sin cos Eq. 1.6
AO AO
We note from the first of the above equation that the normal stress is maximum when = 0; i.e., when the plane
of the section is perpendicular to the axis of the member, and that it approaches zero as approaches 90. Hence,
P
max Eq. 1.7
AO
The second equation shows that the shearing stress is zero for = 0 and = 90, and that for = 45 it reaches
its maximum value
P
max Eq. 1.8
2 AO
1-e. Design Considerations
In engineering applications, the knowledge of stresses is used by engineers to assist in their most important task;
the design of structures and machines that will safely and economically perform a specified function.

1-e.1. Determination of the Ultimate Strength of a Material


An important element to be considered by a designer is how the material that has been selected will behave under
a load. For a given material, this is determined by performing specific tests on prepared samples of the material. For
example, a test specimen of steel may be prepared and placed in a laboratory testing machine to be subjected to a known
centric axial tensile force. As the magnitude of the force is increased, various changes in the specimen are measured; for
example, changes in its length and its diameter. Eventually the largest force which may be applied to the specimen is
reached, and the specimen either breaks or begins to carry less load. This largest force is called the ultimate load for the
test specimen and is denoted by PU. Since the applied load is centric, we may divide the ultimate load by the original
9

cross-sectional area of the rod to obtain the ultimate normal stress of the material used. This stress, also known as the
ultimate strength in tension of the material, is
PU
U Eq. 1.9
A
1-e.2. Allowable Load and Allowable Stress; Factor of Safety
The maximum load that a structural member or a machine component will be allowed to carry under normal
conditions of utilization is considerably smaller than the ultimate load. This smaller load is referred to as the allowable
load and sometimes, as the working load or design load. Thus, only a fraction of the ultimate load capacity of the member
is utilized when the allowable load is applied. The remaining portion of the load-carrying capacity of the member is kept
in reserve to assure its safe performance. The ratio of the ultimate load to the allowable load is used to define the factor of
safety. Hence,
factor of safety = FS =
An alternative definition of the factor of safety is based on the use of stresses;

factor of safety = FS =
This factor of safety includes such factors as the uncertainty of the load, the uncertainty of the material properties, and the
inaccuracy of the stress analysis.

1-e.3. Selection of an Appropriate Factor of Safety


The selection of the factor of safety to be used for various applications is one of the most important engineering
tasks. On the one hand, if a factor of safety is chosen too small, the possibility of failure becomes unacceptably large; on
the other hand, if a factor of safety is chosen unnecessarily large, the result is an uneconomical or nonfunctional design.
The choice of the factor of safety that is appropriate for a given design application requires engineering judgment based
on many considerations such as the following:
1. Variations that may occur in the properties of the member under consideration. The composition, strength, and
dimensions of the member are all subject to small variations during manufacture. In addition, material properties
may be altered and residual stresses introduced through heating or deformation that may occur during manufacture,
storage, transportation, or construction.
2. The number of loadings that may be expected during the life of the structure or machine. For most materials, the
ultimate stress decreases as the number of load applications is increased. This phenomenon is known as fatigue and,
if ignored, may result in sudden failure.
3. The type of loadings that are planned for in the design, or that may occur in the future. Very few loadings are
known with complete accuracy most design loadings are engineering estimates. In addition, future alterations or
changes in usage may introduce changes in the actual loading. Larger factors of safety are also required for
dynamic, cyclic, or impulsive loadings.
4. The type of failure that may occur. Brittle materials fail suddenly, usually with no prior indication that collapse is
imminent. On the other hand, ductile materials, such as structural steel, normally undergo a substantial deformation
called yielding before failing, thus providing a warning that overloading exists. However, most buckling or stability
failures are sudden, whether the material is brittle or not. When the possibility of sudden failure exists, a larger
factor of safety should be used than when failure is preceded by obvious warning signs.
5. Uncertainty due to methods of analysis. All design methods are based on certain simplifying assumptions which
result in calculated stresses being approximations of actual stresses.
6. Deterioration that may occur in the future because of poor maintenance or because of unpreventable natural
causes. A larger factor of safety is necessary in locations where conditions such as corrosion and decay are difficult
to control or even to discover.
7. The importance of a given member to the integrity of the whole structure. Bracing and secondary members may in
many cases be designed with a factor of safety lower than that used for primary members.
10

EXERCISES

1. The 1.4-kip load P is supported by two wooden members of uniform cross section that are joined by the simple
glued scarf shown. Determine the normal and shearing stresses in the glued splice. (Beer, 2012) Ans. = 70 psi;
= 40.4 psi

2. A centric load P is applied to the granite block shown. Knowing that the resulting maximum value of the shearing
stress in the block is 2.5 ksi, determine a) the magnitude of P, b) the orientation of the surface on which the maximum
shearing stress occurs, c) the normal stress exerted on the surface, d) the maximum value of the normal stress in the
block. (Beer, 2009) Ans. a) P = 180 kips b) = 45 c) = 2.5 ksi d) max = 5 ksi

3. A rectangular piece of wood, 50 mm by 100 mm in cross-section, is used as a compression block as shown in the
figure. Determine the maximum axial force P that can be safely applied to the block if the compressive stress in the
wood is limited to 20 MPa and the shearing stress parallel to the grain is limited to 5 MPa. The grain makes an angle
of 20 with the horizontal. (Pytel, 1987) Ans. P = 77.786 kN
P

20

100 mm
11

4. A steel loop ABCD of length 1.2 m and of 10-mm diameter is placed as shown around a 24-mm-diameter aluminum
rod AC. Cables BE and DF, each of 12-mm diameter, are used to apply the load Q. Knowing that the ultimate strength
of the steel used for the loop and the cables is 480 MPa and that the ultimate strength of the aluminum used for the rod
is 260 MPa, determine the largest load Q that can be applied if an overall factor of safety of 3 is desired. (Beer, 2012)
Ans. 15.08 kN

5. In the steel structure shown, a 6-mm-diameter pin is used at C and 10-mm-diameter pins are used at B and D. The
ultimate shearing stress is 150 MPa at all connections, and the ultimate normal stress is 400 MPa in link BD. Knowing
that a factor of safety of 3.0 is desired, determine the largest load P that can be applied at A. Note that link BD is not
reinforced around the pin holes. (Beer, 2012) Ans. 1.683 kN

6. Members AB and BC of the truss shown are made of the same alloy. It is known that a 20-mm square bar of the same
alloy was tested to failure and that an ultimate load of 120 kN was recorded. If a factor of safety of 3.2 is to be
achieved for both bars, determine the required cross-sectional area of bars AB and AC. (Beer, 2006) Ans. AAB = 181.3
mm2; AAC = 213 mm2
12

1-d. Thin-walled Pressure Vessels


A tank carrying a gas or fluid under a pressure p is subjected to tensile forces that resist the bursting forces
developed in the tank. Cylindrical and spherical vessels are the most common geometric shapes that are used as pressure
vessels. A cylindrical pressure vessel may have flat ends so that geometrically, it is a right circular cylinder, or it may
have spherical caps attached to its ends.
Consider a typical longitudinal section A-A through the pressure-loaded cylinder in Fig. 1-8a. A free-body
diagram of the half-cylinder isolated by cutting plane A-A is shown in Fig. 1-5b. The elementary force acting normal to an
element of the cylinder located at an angle from the horizontal diameter is

dF p dA pL D d
2

(a) (b)
Fig. 1-8

A similar force acts on the symmetrically placed element on the other side of the vertical centerline. Since the
horizontal components of such pairs of forces cancel out, the bursting force F is the summation of the vertical components
of these elementary forces:
D
F 0 pL d sin pL cos 0
D
2 2
which reduces to
F pDL
It is apparent that the total bursting force F, acting normal to the cutting plane A-A, is resisted by the equal forces

F 0] F pDL 2 P
v
P acting on each cut surface of the cylinder wall. Applying a vertical summation of forces, we obtain
A simpler method of determining the bursting force F is indicated in Fig. 1.9. Here, the lower half of the cylinder
is occupied by a fluid. Since a fluid transmits pressure equally in all directions, the pressure distribution on the cylinder is
the same as that in Fig. 1-8. From the accompanying free-body diagram, it is apparent that the bursting force F, acting
over that surface of the fluid, equals the pressure intensity p multiplied by the area DL over which it acts, or F = pDL as
before.

(a) (b)
Fig. 1-9
The stress in the longitudinal section that resists the bursting force F is obtained by dividing it by the area of the
two cut surfaces. This gives
pDL
F or t
A 2 tL
13

pD
or t Equation 1.10
2t
This stress is usually called the tangential stress because it acts tangent to the surface of the cylinder. The other common
names are circumferential stress, hoop stress, and girth stress. The stress computed by this formula is the average stress
for cylinders having a wall thickness equal to 1/10 or less of the inner radius.
The free-body diagram of a transverse section is shown in Fig. 1-10. The bursting force acting over the end of the
cylinder is resisted by the resultant P of the tearing forces acting over the transverse section. The area of a transverse
section is the wall thickness multiplied by the mean circumference, or D t t . If t is small compared to D, it is closely
2
D p Dt
4
approximated by Dt. Thus we obtain
pD
or l Equation 1.11
4t
where l denotes what is called the longitudinal stress because it acts parallel to the longitudinal axis of the cylinder.

Fig. 1-10
Comparing the two equations for stress shows that the longitudinal stress is one-half the value of the tangential
stress. This means that if the pressure in the cylinder is raised to the bursting point, failure will occur along a longitudinal
section or longitudinal seam of the cylinder. Thus, in design, the permissible internal pressure will depend on the strength
of the longitudinal joint. When the ends of the cylinder are rounded or dished, the bursting force on the transverse section
is still equal to the product of the internal pressure multiplied by the projected area of the transverse section.

EXERCISES:

1. A cylindrical steel pressure vessel 400 mm in diameter with a wall thickness of 20 mm is subjected to an internal
pressure of 4.5 MPa. a) Calculate the tangential and longitudinal stresses in the steel. b) To what value may the
internal pressure be increased if the stress in the steel is limited to 120 MPa? (Pytel, 1987) Ans. a) t = 45 MPa; l =
22.5 MPa; b) 12 MPa
5
2. The wall thickness of a 4-ft diameter spherical tank is in. Calculate the allowable internal pressure if the stress is
16
limited to 8000 psi. (Pytel, 1987) Ans. 208.333 psi

3. A water tank, 22 ft in diameter, is made from steel plates that are 5 in. thick. Find the maximum height to which the
16
tank may be filled if the circumferential stress is limited to 6000 psi. (Pytel, 1987) Ans. h = 52.448 ft

4. The strength of the longitudinal joint of the tank in the figure is 33 kips/ft, whereas for the girth joint it is 16 kips/ft.
Calculate the maximum diameter of the cylindrical tank if the internal pressure is 150 psi. (Pytel, 1987) Ans. 35.56 in.
14

5. The tank shown is fabricated from -in steel plate. Calculate the maximum longitudinal and circumferential stresses
caused by an internal pressure of 125 psi. (Pytel, 1987) Ans. l = 6566.023 psi; t = 252 ksi

6. A pipe carrying steam at 3.5 MPa has an outside diameter of 450 mm and a wall thickness of 10 mm. A gasket is
inserted between the flange at one end of the pipe and a flat plate used to cap the end. How many 40-mm diameter
bolts must be used to hold the cap on if the allowable stress in the bolts is 80 MPa, of which 55 MPa is the initial
stress? What circumferential stress is developed in the pipe? (Pytel, 1987) Ans. 17 bolts; 75.25 MPa
15

CHAPTER 2 SIMPLE STRAIN

2-a. Introduction
The strength of the material is not the only criterion that must be considered in designing structures. The stiffness
of a material is of equal importance. The mechanical properties such as hardness, toughness and ductility determine the
selection of a material. Strain can be thought of as a measure of the deformation characteristics of a load-carrying
member. Any structure will deform when subject to an external load; hence, strain is always associated with stress.
An axially loaded member undergoes a change in length, becoming longer when in tension and shorter when in
compression. The change in length, called normal or linear strain, is a measure of deformation per unit length of a
member. To obtain the unit deformation or strain, denoted by the Greek letter , we divide the elongation by the length
L in which it was measured, thereby obtaining
Equation 2.1
L
The strain so computed, however, measures only the average value of strain. The correct expression for strain at any
position is
d
dL
where d is the differential elongation of the differential length dL. However, under certain conditions, the strain may be
assumed constant and its value computed from the equation for average stress for the following conditions:
1. The specimen must be of constant cross section.
2. The material must be homogeneous.
3. The load must be axial, that is, produce uniform stress.
Because normal strain is a ratio of two lengths, it is a dimensionless quantity. However, in practice, it is common to use
units of meters per meter or inches per inch.

2-b. Stress-Strain Diagram


The mechanical properties of materials used in engineering are determined by tests performed on small specimens
of the material. The tests are conducted in materials-testing laboratories equipped with testing machines capable of
loading the specimens in a variety of ways.
The most common materials test is the tension test, in which tensile loads are applied to a cylindrical specimen.
The ends of the specimen are gripped between the jaws of the testing machine. Values of the load and the elongation in a
specified length, called the gage length, are observed simultaneously. These data are then plotted on a graph with the
ordinate representing the load and the abscissa representing the elongation.

Fig. 2-1
Fig. 2-1 shows the stress-strain diagram of a typical structural steel in tension. Strains are plotted on the horizontal
axis and stresses on the vertical axis. The diagram begins with a straight line from the origin O to point A, which means
that the stress and strain are proportional. Beyond point A, the proportionality between stress and strain no longer exists;
hence the stress at A is called the proportional limit. The slope of the straight line from O to A is called the modulus of
elasticity. Because stain is non-dimensional, this slope has the same units as stress. With an increase in the load beyond
the proportional limit, the strain begins to increase more rapidly for each increment in stress. The slope of the stress-strain
curve then becomes smaller and smaller, until, at point B, the curve becomes horizontal. Beginning at this point,
16

considerable elongation occurs, with no noticeable increase in the tensile force (from B to C). This phenomenon is known
as yielding of the material, and point B is called the yield point. The corresponding stress is known as the yield stress of
the steel. In the region from B to C, the material becomes perfectly plastic, which means that it can deform without an
increase in the applied load.
After undergoing the large strains that occur during yielding in the region BC, the steel begins to strain harden.
During strain hardening, the material undergoes changes in its atomic and crystalline structure, resulting in increased
resistance of the material to further deformation. Additional elongation now requires an increase in tensile load and the
stress-strain diagram has a positive slope from C to D. The load eventually reaches its maximum value and the
corresponding stress at point D is called the ultimate stress. Further stretching of the bar is actually accompanied by a
reduction in the load, and fracture finally occurs at a point such as E, known as rupture strength.
Lateral contraction of the specimen occurs when it is stretched, resulting in a decrease in the cross-sectional area.
The reduction in area is too small to have a noticeable effect on the calculated value of stress up to about point C, but
beyond that point the reduction begins to alter the shape of the diagram. The true stress is larger than the nominal stress
because it is calculated with a smaller area. In the vicinity of the ultimate stress, the reduction in area of the bar becomes
clearly visible and a pronounced necking of the bar occurs. If the actual cross-sectional area at the narrow part of the neck
is used to calculate the stress, the true stress-strain curve will follow the dashed line CE. The total load the bar can carry
diminishes after the ultimate stress is reached (curve DE), but this reduction is due to the decrease in area of the bar and
not to a loss in strength of the material itself. In reality, the material withstands an increase in stress up to failure (point
E).

2-c. Hookes Law: Axial and Shearing Deformations


Most structural materials have an initial region on the stress-strain diagram in which the material behaves both
elastically and linearly (the region from the origin O up to the proportional limit at point A in Fig. 2-1). When a material
behaves elastically and also exhibits a linear relationship between stress and strain, it is said to be linearly elastic. This
type of behavior is extremely important in engineering because many structures and machines are designed to function at
low levels of stress in order to avoid permanent deformations from yielding or plastic flow. Linear elasticity in the initial
region of the stress-strain diagram is a property of many solid materials including metals, wood, concrete, plastics, and
ceramics.
The linear relationship between stress and strain for a bar in simple tension or compression can be expressed by
the equation
E
in which E is a constant of proportionality known as the modulus of elasticity for the material. The modulus of elasticity is
the slope of the stress-strain diagram in the linearly elastic region, and its value depends upon the particular material being
used.
The equation = E is commonly known as Hookes Law, named after the famous English scientist Robert
Hooke who first investigated the elastic properties of materials, thus establishing the linear relationship between the
applied load and the resulting elongation. Another English scientist, Thomas Young, introduced the constant of
proportionality E that came to be known as Youngs modulus which eventually was called modulus of elasticity.
A convenient variation of Hookes law is obtained by replacing by its equivalent P and replacing by
A L
resulting to the equation
PL or L Equation 2.2
AE E
This equation is subject to all restrictions previously discussed in connection with the equation for axial deformation
which are
1. The load must be axial.
2. The bar must have a constant cross section and be homogeneous.
3. The stress must not exceed the proportional limit.
Shearing forces cause a shearing deformation in which the element subject to shear undergoes a change in shape
from a rectangle to a parallelogram, as shown in Fig. 2-2. The action may be visualized as equivalent to the infinitesimal
sliding and infinitely thin layers past each other, thereby resulting in the total shearing deformation s in the length L.
The average shearing strain, , is found by dividing s by L; i. e.,
s

L
17

Ps s

Ps
Fig. 2-2
The relation between shearing stress and shearing strain, assuming Hookes law to apply to shear, is
= G
in which G represents the modulus of rigidity. The relation between the shearing deformation and applied shearing forces
is expressed by
VL
s
As G
in which V is the shearing force acting over the shearing area As.

EXERCISES

1. A uniform bar of length L, cross-sectional A, and a unit mass is suspended vertically from one end. Show that its
gL2 MgL
total elongation is . If the total mass of the bar is M, show also that .
2E 2 AE

2. A steel rod having a cross-sectional area of 300 mm2 and a length of 150 m is suspended vertically from one end. It
supports a tensile load of 20 kN at the lower end. If the unit mass of steel is 7850 kg/m3 and E = 200 103 MPa, find
the total elongation of the rod. (Pytel, 1987) Ans. 54.33 mm

3. An 18-m-long steel wire of 5-mm diameter is to be used in the manufacture of a prestressed concrete beam. It is
observed that the wire stretches 45 mm when a tensile force P is applied. Knowing that E = 200 GPa, determine (a)
the magnitude of the force P, (b) the corresponding normal stress in the wire. (Beer, 2012) Ans. a) 9.82 kN
(b) 500 MPa

4. A homogeneous prismatic bar is loaded by forces P1, P2, and P3 acting at points B, C, and D, respectively. The
dimensions of the bar are as follows: a = 1.0 m, b = 0.4 m, and c = 0.6 m. Assuming that P2 = 15 kN and P3 = 9 kN,
determine the force P1 so that the lower end D of the bar does not move vertically when the loads are applied. (Gere,
1991) Ans. 39 kN

a
P1

B
b
C

c
P2
D

P3
18

5. A bronze bar is fastened between a steel bar and an aluminum bar as shown in the figure. Axial loads are applied at
the points indicated. Find the largest value of P that will not exceed an overall deformation of 3.0 mm or the
following stresses: 140 MPa in the steel, 120 MPa in the bronze, and 80 MPa in the aluminum. Assume that the
assembly is suitably braced to prevent buckling. Use Es = 200 GPa, Ea = 70 GPa, and Eb = 83 GPa. (Pytel, 1987)
Ans. 12.8 kN
Bronze
Steel Aluminum
A = 650 mm2
P A = 480 mm2 A = 320 mm2 2P
3P 4P
1.0 m 2.0 m 1.5 m

6. The rigid bar ABC shown is hinged at A and supported by a steel rod at B. Determine the largest load P that can be
applied at C if the stress in the steel rod is limited to 30 ksi and the vertical movement of end C must not exceed 0.10
in. (Pytel, 1987) Ans. 4833.33 lb

Steel
L = 4 ft
A = 0.50 in2
E = 29 106 psi


A B C
2 ft 3 ft
P

7. The rigid bar AB, attached to vertical rods as shown in the figure, is horizontal before the load P is applied. Determine
the vertical movement of P if its magnitude is 50 kN. (Pytel, 1987) Ans. 1.878 mm

Steel
L=4m
Aluminum A = 300 mm2
L=3m E = 200 GPa
A = 500 mm2
E = 70 GPa

A B
3.5 m 2.5 m
P

8. The rigid bars AB and CD shown are supported by pins at A and C and the two rods. Determine the maximum force P
that can be applied as shown if its vertical movement is limited to 5 mm. Neglect the weights of the members. (Pytel,
1987) Ans. 76.364 kN

Aluminum
L=2m
A = 500 mm2
E = 70 GPa
A 3m 3m B

Steel
L=2m
A = 300 mm2
E = 200 GPa
D
C

P
19

9. As shown in the figure, two aluminum rods AB and BC, hinged to rigid supports, are pinned together at B to carry a
vertical load P = 6000 lb. If each rod has a cross-sectional area of 0.60 in2 and E = 10 106 psi, compute the
elongation of each rod and the horizontal and vertical displacements of point B. (Pytel, 1987) Ans. H = 0.0277 in.;
V = 0.192 in.

10 ft

30
B
30

6 ft
P C

10. The steel bars AB and BC are pinned at each end and support the load of 200 kN as shown in the figure. The material
is structural steel having a yield stress of 200 MPa and safety factors of 2 and 3.5 for tension and compression,
respectively. Determine the size of each bar and also the horizontal and vertical components of displacement of point
B. Take E = 200 GPa. Neglect any possibility of lateral buckling of bar BC. Ans. AAB = 1732 mm2; ABC = 1750 mm2;
h = 0.37 mm ; v = 1.78 mm

200 kN

B
60

C 2m
20

2-d. Statically Indeterminate Members


There are certain combinations of axially loaded members in which the equations of static equilibrium are not
sufficient for a solution. This condition exists in structures where the reactive forces or the internal resisting forces over a
cross section exceed the number of independent equations for equilibrium. Such cases are called statically indeterminate
and require the use of additional relations that depend on the elastic deformations of the members; i.e., additional
geometric relations between the elastic deformations produced by the loads.

EXERCISES

1. The 1.5 m concrete post is reinforced with six steel bars, each with a 28-mm diameter. Determine the maximum
centric force P that may be applied if the allowable normal stress is 160 MPa in the steel and 18 MPa in the
concrete. Use Es = 200 GPa and Ec = 25 GPa. (Beer, 2006) Ans. 3330 kN

2. Two cylindrical rods, one of steel and the other of brass, are joined at C and restrained by rigid supports at A and E.
For the loading shown and knowing that Es = 200 GPa and Eb = 105 GPa, determine a) the reactions at A and E, b) the
deflection of point C. (Beer, 2012) Ans. a) RA = 62.8 kN ; RE = 37.2 kN b) 46.3 103 mm

3. The composite rod in the figure is stress-free before the axial loads P1 and P2 are applied. Assuming that the right wall
yields 0.80 mm, calculate the stress in each material if P1= 150 kN and P2 = 90 kN. (Pytel, 1987) Ans. al =12.617
MPa; st = 69.322 MPa; br =190.537 MPa
Steel Bronze
Aluminum A =2000 mm2
A = 900 mm2 A = 1200 mm2
E = 200 GPa E = 83 GPa
E = 70 GPa
P1 P2

500 mm 250 mm 350 mm

4. Three steel eye-bars, each 4 in. by 1 in. in section, are to be assembled by driving rigid in. diameter drift pins
through holes drilled in the ends of the bars. The centerline spacing between the holes is 30 ft in the two outer bars,
but is 0.045 in. shorter in the middle bar. Find the shearing stress developed in the drift pins. Neglect local
deformation at the holes. (Pytel, 1987) Ans. 8038.536 psi
21

5. The rigid platform in the figure has negligible mass and rests on two steel bars, each 250 mm long. The center bar is
aluminum and 249.90 mm long. Compute the stress in the aluminum bar after the center load P = 400 kN has been
applied. For each steel bar, the area is 1200 mm2 and E = 200 GPa. For the aluminum bar, the area is 2400 mm2 and
E = 70 GPa. (Pytel, 1987) Ans. 22.476 MPa

Steel Aluminum Steel

6. The assembly in the figure consists of a light rigid bar AB, pinned at O, that is attached to the steel and aluminum
rods. In the position shown, bar AB is horizontal and there is a gap = 5 mm between the lower end of the steel rod
and its pin support at C. Compute the stress in the aluminum rod when the lower end of the steel rod is attached to its
support. (Pytel, 1987) Ans. 130.802 MPa

0.75 m 1.5 m

A B

Steel Aluminum
A =250 mm2 A = 300 mm2
E = 200 GPa E = 70 GPa
L=2m


C D

7. The length of the 2-mm diameter steel wire CD has been adjusted so that with no load applied, a gap of 1.5 mm exists
between the end B of the rigid beam ACB and a contact point E. Knowing that E = 200 GPa, determine where a 20-kg
block should be placed on the beam in order to cause contact between B and E. (Beer, 2009) Ans. 92.6 mm
22

8. The light rigid bar ABCD shown in the figure is pinned at B and connected to two vertical rods. Assuming that the bar
was initially horizontal and the rods stress-free, determine the stress in each rod after the load P = 20 kips is applied.
(Pytel, 1987) Ans. s = 31.35 ksi; a = 16.217 ksi

Steel
L = 3 ft
A = 0.5 in2
E = 29 106 psi
A B C D

Aluminum
L = 4 ft
A = 0.75 in2
P
E = 10 106 psi 2 ft
4 ft 2 ft

9. As shown in the figure, a rigid bar with negligible mass is pinned at O and attached to two vertical rods. Assuming
that the rods were initially stress-free, what maximum load P can be applied without exceeding stresses of 150 MPa
in the steel rod and 70 MPa in the bronze rod? (Pytel, 1987) Ans. 107.4 kN

2m 1.5 m 1.5 m

Bronze
Steel A = 300 mm2
A = 900 mm2 E = 83 GPa
E = 200 GPa L=2m
P L = 1.5 m

10. Three bars AB, AC, and AD are pinned together as shown in the figure. Initially, the assembly is stress-free.
Horizontal movement of the joint at A is prevented by the short horizontal strut AE. Calculate the stress in each bar
and the force in the strut AE when the assembly is used to support the load W = 10 kips. For each steel bar, A = 0.3 in2
and E = 29 106 psi. For the aluminum bar, A = 0.6 in2 and E = 10 106 psi. (Pytel, 1987) Ans. AB = 9.923 ksi;
AC = 5.83 ksi; AD = 14.97 ksi; PAE = 378 lb

B C D

Aluminum
10 ft Steel Steel

40 20

A
E

W
23

2-e. Poissons Ratio: Biaxial and Triaxial Deformations


Another type of elastic deformation is the change in transverse dimensions accompanying axial tension or
compression. Experiments show that if a bar is lengthened by axial tension, there is a reduction in the transverse
dimensions. Simeon D. Poisson showed that the ratio of the unit deformations or strains in these directions is constant for
stresses within the proportional limit. This ratio is named after him; it is denoted by and defined by
z y

x x
where x is the strain due only to stress in the x direction and y and z are the strains induced in the perpendicular
directions. The minus sign indicates a decrease in transverse dimensions when x is positive, as in the case of tensile
elongation.
Poissons ratio permits us to extend Hookes law of uniaxial stress to the case of biaxial stress. Thus if an element
is subjected simultaneously to tensile stresses in the x and y directions, the strain in the x direction due to the tensile stress
x y
x is . Simultaneously the tensile stress y will produce lateral contraction in the x direction of the amount v , so the
E E
resultant unit deformation or strain in the x direction will be
x y
x
E E
Similarly, the total strain in the y direction is
y x
y
E E
If desired, these two equations can be solved to express the stresses in terms of the strains as follows:

x
x y E ; y
y x E
1 2
1 2

A further extension of this discussion results in the following expressions for strains caused by the simultaneous
action of triaxial tensile stresses:
x 1 x y z
E

y 1 y x z Equation 2.3
E
z
1

z x y
E
All these equations are valid for compressive effects also; it is only necessary to assign positive signs to elongations and
tensile stresses, and conversely, negative signs to contractions and compressive stresses.
An important relation among the constants for a given material is expressed by
G E
2 1
which is useful for computing values of when E and G have been determined.
24

EXERCISES

1. A high-strength steel rod with E = 200 GPa and = 0.3 is compressed by an axial force P. When there is no axial
load, the diameter of the rod is exactly 50 mm. In order to maintain certain clearances, the diameter of the rod must
not exceed 50.025 mm under load. What is the largest permissible load P? (Gere, 1991) Ans. 654.5 kN
50 mm

P P

2. A thin plate is in a state of plane stress and has dimensions of 8 in. in the x direction and 4 in. in the y direction. The
plate increases in length in the x direction by 0.0016 in. and decreases in the y direction by 0.00024 in. Compute x
and y to cause these deformations. E = 29 106 psi and v = 0.30. (Buchanan, 1988) Ans. x = 5800 psi; y = 0
y

4 x

3. A 150-mm long bronze tube, closed at its ends, is 80 mm in diameter and has a wall thickness of 3 mm. It fits without
clearance in an 80-mm hole in a rigid block. The tube is then subjected to an internal pressure of 4 MPa. Assuming
v = and E = 83 GPa, determine the tangential stress in the tube. (Pytel, 1987) Ans. 8.889 MPa
y

150 mm x

80 mm

4. The block shown is made of a magnesium alloy for which E = 45 GPa and v = 0.35. Knowing that x = 140 MPa,
determine a) the magnitude of y for which the change in the height of the block will be zero; b) the corresponding
change in the area of the face ABCD; c) the corresponding change in the volume of the block. (Beer, 2006)
Ans. a) 49 MPa; b) 3.15 106 m2; c) 110.25 109 m3

x
25

5. A rectangular steel block is 3 in. long in the x direction, 2 in. long in the y direction, and 4 in. long in the z direction.
The block is subjected to triaxial loading consisting of three uniformly distributed forces as follows: 48 kips tension in
the x direction, 60 kips compression in the y direction, and 54 kips tension in the z direction. If v = 0.30 and E = 29
106 psi, determine the single uniformly distributed load in the x direction that would produce the same deformation in
the y direction as the original loading. (Pytel, 1987) Ans. 654.5 kN
z

Pz

Px
4
Py Py
y

Px

3
x
2
Pz

2-f. Thermal Stresses


All engineering materials are affected by temperature change. Changes in temperature cause bodies to expand or
contract because common materials expand when heated and contract when cooled. This phenomenon is characterized by
the coefficient of thermal expansion, . For a given material, is the strain due to a 1-degree temperature change. The
amount of linear deformation, T is expressed by the relation
T = L(T) Equation 2.4
in which L is the length and T is the temperature change.
If a temperature deformation is permitted to occur freely as by the use of expansion joints, no load or stress will
be induced in the structure. But in some cases, it may not be feasible to permit these temperature deformations; the result
is that internal forces are created that resist them. The stresses caused by these internal forces are known as thermal
stresses.

EXERCISES
1. A steel rod with a cross-sectional area of 0.25 in2 is stretched between two fixed points. The tensile load at 70F is
1200 lb. What will be the stress at 0F? At what temperature will the stress be zero? Assume = 6.5 106 in/(in-F)
and E = 29 106 psi. (Pytel, 1987) Ans. 17995 psi; 95.46 F

2. Calculate the increase in stress for each segment of the compound bar shown if the temperature increases by 100F.
Assume that the supports are unyielding and that the bar is suitably braced to prevent buckling. (Pytel, 1987)
Ans. a = 13345.918 psi; s = 17794.558 psi
Aluminum Steel
A = 2 in2 A = 1.5 in2
E = 10 106 psi E = 29 106 psi
= 12.8 106 /F = 6.5 106 /F

10 in. 15 in.
26

3. The rigid bar ABC is pinned at B and attached to the two vertical rods. Initially, the bar is horizontal and the vertical
rods are stress-free. Determine the stress in the aluminum rod if the temperature of the steel rod is decreased by 40C.
Neglect the weight of bar ABC. (Pytel, 1987) Ans. 49.14 MPa

Aluminum
Steel L = 1.2 m
L = 0.9 m A = 1200 mm2
A = 300 mm2 E = 70 109 Pa
E = 200 109 Pa = 23 m/(m-C)
= 11.7 m/(m-C)
A B C

0.6 m 1.2 m

4. A 1.2-m concrete post is reinforced by four steel bars, each of 18-mm diameter. Knowing that Es = 200 GPa, s = 11.7
10-6/C and Ec = 25 GPa, c = 9.9 10-6/C, determine the normal stresses induced in the steel and in the concrete by
a temperature rise of 27C. (Beer, 2006) Ans. st = 8.04 MPa; c = 0.21 MPa

5. At room temperature (20C) a 0.5 mm gap exists between the ends of the rods shown. At a later time when the
temperature has reached 140C, determine a) the normal stress in the aluminum rod, b) the change in length of the
aluminum rod. (Beer, 2009) Ans. a) 116.2 MPa; b) 0.363 mm

Aluminum Steel
A = 2000 mm2 A = 800 mm2
E = 75 GPa E = 190 GPa
= 23 106 /C = 17.3 106 /C
27

CHAPTER 3 TORSION

3-a. Introduction
Torsion refers to the twisting of a structural member when it is loaded by couples that produce rotation about its
longitudinal axis. The moment of the couple is equal to the product of one of the forces and the perpendicular distance
between the lines of action of the forces. Torsion occurs in many physical situations. For instance, the drive shaft of a
vehicle, while it delivers power from one point to another, is being stressed by a torsional moment. Many times a system
of pulleys might be attached to a common drive shaft and will serve to input various torques to the drive shaft.
The formulas that will be developed in this topic is limited to the stresses and deformations produced in circular
bars subjected to torsion. Examples of such bars are axles and drive shafts in machinery, propeller shafts, drill rods,
screwdrivers, and steering rods.

3-b. Derivation of Torsion Formulas


Consider a bar or shaft of circular cross section twisted by couples T acting at the ends. From considerations of
symmetry, we can show that the cross sections of the circular bars rotate as rigid bodies about the longitudinal axis, with
radii remaining straight and the cross sections remaining plane and circular.
During twisting, there will be a rotation about the longitudinal axis of one end of the bar with respect to the other.
In Fig. 3-1, a fiber AB on the outside surface, which is originally straight, will be twisted into a helix AC as the shaft is
twisted through the angle .

Fig. 3-1
Consider any internal fiber located a radial distance from the axis of the shaft. Since the projection upon a
transverse section of straight radial lines in the section remains straight, the radius of such a fiber also rotates through the
angle , causing a total shearing deformation s equal to DE. The length of this deformation is the arc of a circle whose
radius is and which is subtended by the angle of radians; the length is given by
s = DE =
The unit deformation of this fiber is

s

L L
The shearing stress at this typical fiber is determined from Hookes law to be

G G Equation 3.1
L
This equation may be called the equation of compatibility, since the stresses expressed by it are compatible with the
elastic deformations. Note that each of the terms in the parentheses in this equation is a constant that does not depend on
the particular internal fiber chosen for analysis; the product of these terms represents a constant. Therefore we conclude
that the shearing stress at any internal fiber is determined by the product of a constant and a variable radial distance; that
is, the stress distribution along any radius varies linearly with the radial distance from the axis of the shaft. For the stress
variation along the radius OB, the maximum stress occurs at the outside fiber.
Figure 3-2 shows the free-body diagram of the left portion when the shaft is divided into two segments by the
cutting plane M-N.
A differential area of section M-N at a radial distance from the axis of the shaft carries the differential resisting
load dP = dA. By considering an area infinitesimally small, we may assume the stress to be uniform over such area.
Since the function of this resisting load dP is to produce resistance to the applied torque T, the load must be directed
perpendicular to the radius in order to produce the maximum effect.
28

Fig. 3-2

To satisfy the condition of static equilibrium, we apply M = 0, or the fact that the applied torque T is equal to the
resisting torque Tr. The resisting torque Tr is the sum of the resisting torques developed by all differential loads dP.
T Tr dP dA
Replacing by its value from Equation 3.1 gives
G
T
L
2 dA

or, since 2 dA = J, the polar moment of inertia of the cross section,


G
T J
L
This is usually written
TL Equation 3.2
JG
By replacing the product G /L in Equation 3.1 by its equivalent value T/J from Equation 3.2, we obtain
T
Equation 3.3
J
Equation 3.3 is called the torsion formula.
The formula that determines the maximum shearing stress is a more common form of the torsion formula. It is
obtained by replacing by the radius r of the shaft:
Max . Tr Equation 3.4
J
Note that since Hookes law was used in deriving these equations, the stresses must not exceed the proportional limit;
also, these formulas are applicable only to circular shafts, either solid or hollow. The values of polar moments of inertia
for circular shafts are the following:

Solid shaft: Hollow shaft:

In many practical applications, shafts are used to transmit power. From dynamics, it is known that the power P
transmitted by a constant torque T rotating at a constant angular speed is given by
= T
where is measured in radians per unit time. If the shaft is rotating with a frequency of f revolutions per unit time, = 2
f, and we have
P = 2 f T Equation 3.5
P
Thus the torque can be expressed as T
2f
29

EXERCISES

1. A steel shaft 3 feet long that has a diameter of 4 in. is subjected to a torque of 15 ft-kips. Determine the maximum
shearing stress and the angle of twist. Use G = 12 106 psi. (Pytel, 1987) Ans. = 14.324 ksi; = 1.23

2. A solid steel shaft 5 m long is stressed to 80 MPa when twisted through 4. Using G = 83 GPa, compute the shaft
diameter. What power can be transmitted by the shaft at 20 Hz?. (Pytel, 1987) Ans. 5.195 MW

3. The electric motor exerts a 500 N-m torque on the aluminum shaft ABCD when it is rotating at a constant speed.
Knowing that G = 27 GPa and that the torques exerted on pulleys B and C are as shown, determine the angle of twist
between B and D. (Beer, 2006) Ans. 3.22

300 N-m

200 N-m

4. An aluminum shaft with constant diameter of 50 mm is loaded by torques applied to gears attached to it as shown in
the figure. Using G = 28 GPa, determine the relative angle of twist of gear D relative to gear A. (Pytel, 1987)
Ans. 6.33

5. A compound shaft consisting of a steel segment and an aluminum segment is acted upon by two torques as shown in
the figure. Determine the maximum permissible value of T subject to the following conditions; s 83 MPa, a 55
MPa, and the angle of rotation of the free end is limited to 6. For steel, G = 83 GPa and for aluminum, G = 28 GPa. .
(Pytel, 1987) Ans. 679.042 N-m
30

6. The compound shaft shown in the figure is attached to rigid supports. For the bronze segment AB, the maximum
shearing stress is limited to 8000 psi and for the steel segment BC, it is limited to 12 ksi. Determine the diameters of
each segment so that each material will be simultaneously stretched to its permissible limit when a torque T = 12 ft-
kips is applied. For bronze, G = 6 106 psi and for steel, G = 12 106 psi. (Pytel, 1987) Ans. dbr = 4.258 in.;
dst = 2.129 in.
T
Bronze
Steel
B
A C
6 ft 4 ft

7. Two solid steel shafts are connected by the gears shown. A torque of magnitude T = 900 N-m is applied to shaft AB.
Knowing that the allowable shearing stress is 50 MPa and considering only stresses due to twisting, determine the
required diameter of shafts AB and CD. (Beer, 2006) Ans. dAB = 45.1 mm; dCD = 65.0 mm

T
8. In a rivet group subjected to a twisting couple T, show that the torsion formula = can be used to find the
J
shearing stress at the center of any rivet. Let J = A2, where A is the area of a rivet at the radial distance from
the centroid of the rivet group. (Pytel, 1987)

9. Six -in-diameter rivets fasten the plate in the figure to the fixed member. Determine the average shearing stress
caused in each rivet by the 14-kip loads. (Pytel, 1987) Ans. max = 13.991 ksi; min = 7.761 ksi
31

10. A plate is fastened to a fixed member by four 20-mm diameter rivets arranged as shown. Compute the maximum and
minimum shearing stress developed. (Pytel, 1987) Ans. max = 40.107 MPa; min = 13.369 MPa

11. The plate shown in the figure is fastened to the fixed member by five 10-mm-diameter rivets. Compute the value of
the loads P so that the average shearing stress in any rivet does not exceed 70 MPa. (Pytel, 1987) Ans. 8915.025 N
32

CHAPTER 4 SHEAR FORCES AND BENDING MOMENTS IN BEAMS

4-a. Introduction
The beam is one of the fundamental structural members. A beam is a structural member supporting loads applied
at various points along the member. Beams are usually long, straight prismatic members that span the distance between
supports and is loaded primarily in a transverse direction. Such a transverse loading causes only bending and shear in the
beam. When the loads are not at a right angle to the beam, they also produce axial forces in the beam.
Beams are generally classified according to support conditions. There are two primary classifications for support
conditions in two dimensions: simple support and fixed support. Simple supports may be categorized further. Fig. 4-1
illustrates the simple support conditions. In Figs. 4-1a and 4-1b, the roller-type simple support, the reaction is illustrated
normal to the supporting surface. In Fig. 4-1c, the reaction would lie along the axis of the link. These support conditions
require the evaluation of only one unknown. The roller support allows the structure to actually move in a direction normal
to the reaction. Fig. 4-1d shows a second type of simple support, referred to as a pin or hinge. The pin will resist
translation in any direction, hence has two unknown components of reaction, usually vertical and horizontal. Both the
roller and the pin will allow the beam to rotate.

(a) (b) (c)

(d)
Fig. 4-1
The fixed support is illustrated in Fig. 4-2. It will resist movement in any direction in the plane and will also resist
rotation of the beam. The fixed support may also be referred to as clamped or cantilevered, and will resist two components
of displacement and any rotation of the support; hence there are two force components and one moment component.

RH

M
RV Fig. 4-2
The types of beams according to how they are supported are generally classified as statically determinate and
statically indeterminate beams. The beams shown in Fig. 4-3 are statically determinate and their reactions can be
determined from equilibrium equations. A simple beam, Fig. 4-3a, is supported by a hinged reaction at one end and a
roller support at the other. An overhanging beam (Fig. 4-3b) is supported by a hinge and a roller reaction, with either or
both ends extending beyond the supports. A cantilever beam (Fig. 4-3c) is supported at one end only, with a suitable
restraint to prevent rotation of that end.

Fig. 4-3

Other methods of supporting beams are shown in Fig. 4-4. Such beams are statically indeterminate. The
continuous beam (Fig. 4-4a), the propped beam (Fig. 4-4b), and the fixed-ended or restrained beam (Fig. 4-4bc) all have
at least one or more reactive element than is absolutely necessary to support them. The presence of excess supports
requires the use of additional equations obtained from considering the elastic deformations of the beam.
33

(a) (b) (c)

Fig. 4-4
Sometimes two or more beams are connected by hinges to form a single continuous structure (compound beams),
such as the beams shown in Fig. 4-5. In both beams, the reactions at the supports involve four unknowns and cannot be
determined from the free-body diagram of the compound beam. They can be determined by considering the free-body
diagram of each beam separately.

Fig. 4-5
Loads acting on beams may be of several kinds, as illustrated in Fig. 4-6. A concentrated load is any type of
loading that can be idealized as a single force acting at a point, such as P1, P2, P3, and P4. Distributed loads act over a
considerable length of the beam, as shown in Figs. 4-6a and 4-6b. Such loads are measured by their intensity, which is
expressed in units of force per unit distance along the axis of the beam. A uniformly distributed load or uniform load, Fig.
4-6a, has constant intensity per unit distance. A varying load has an intensity that changes with distance along the axis, as
in Fig. 4-6b. Another kind of load is a couple, illustrated by the couple moment M1 acting on the overhanging beam of
Fig. 4-6c.

(a) (b) (c)


Fig. 4-6

When analyzing beams, it is often necessary to distinguish between pure bending and nonuniform bending. Pure
bending refers to flexure of a beam under a constant bending moment . Therefore, pure bending occurs only in regions of
a beam where the shear force is zero. In contrast, nonuniform bending refers to flexure in the presence of shear forces,
which means that the bending moment changes as we move along the axis of the beam.

4-b. Shear and Moment


When a beam is loaded by forces or couples, internal stresses and strains are created. To determine these stresses
and strains, we first must find the internal forces and internal couples that act on cross sections of the beam.

(a) (b) (c)


Fig. 4-7
Fig. 4-7a shows a simple beam that carries a concentrated load P and is held in equilibrium by the reactions R1
and R2 . For the time being, neglect the mass of the beam itself and consider only the effect of the load P. Assume that a
cutting plane a-a at a distance x from R1 divides the beam into two segments. The free-body diagram of the left segment in
Fig. 4-7b shows that the externally applied load is R1. To maintain equilibrium of this segment of the beam, the fibers in
the exploratory section a-a must supply the resisting forces necessary to satisfy the conditions of static equilibrium. In this
case, the external load is vertical, so the condition Fx = 0 is automatically satisfied. To satisfy Fy = 0, the vertical
unbalance caused by R1 requires the fibers in section a-a to create a resisting force. This is shown as Vr, and is called the
resisting shearing force. For the loading shown, Vr is numerically equal to R1; but if additional loads had been applied
34

between R1 and section a-a, the net vertical unbalance would be found from the summation of their vertical components.
We define this net vertical unbalance as the shearing force in the beam. It is denoted by V and may be determined from
the summation of the vertical components of the external loads acting on either side of the section. This definition of
shearing force (also called vertical shear or shear) may be expressed mathematically as
V = (Fy )L = (Fy )R
the subscript L emphasizing that the vertical summation includes only the external loads acting on the beam segment to
the left of the section being considered and the subscript R referring to the right of the section being considered. The
resisting shear Vr set up by the fibers in any section is always equal but oppositely directed to the shearing force V.
For complete equilibrium of the free-body diagram in Fig. 4-7b, the summation of moments must also balance. In
this discussion, R1 and Vr are equal, thereby producing a couple M that is equal to R1x and is called the bending moment
because it tends to bend the beam. The fibers in the exploratory section must create a numerically equal resisting moment
Mr, that acts as shown.
Bending moment is defined as the summation of moments about the centroidal axis of any selected section of all
the loads acting either to the left or to the right side of the section, and is expressed mathematically as
M = (My )L = (My )R
the subscript L indicating that the bending moment is computed in terms of the loads acting to the left of the section and
the subscript R referring to loads to the right of the section.
Sign Conventions

Fig. 4-8

The following are the interpretations of Fig. 4-8 for the sign conventions for shear and bending moment:
1. The shear at any given point of a beam is positive when the external forces (loads and reactions) acting on the beam
tend to move up the left portion of the beam past the right portion, or if the external forces tend to move down the
right portion past the left portion.
2. The bending moment at any given point of a beam is positive when the external forces acting on the beam tend to
bend the beam upward.
Insofar as the left segment of a beam is concerned, Fig. 4-7b, in computing V, upward acting forces or loads are
considered positive. For bending moment, it is equivalent to taking clockwise moments about the bending axis as positive.
With respect to the right segment of a beam, Fig. 4-7c, this convention means that the shearing force is positive if the
force is downward and the moment is positive in a counterclockwise direction.

4-b.1. Shear and Moment Diagrams


The shear forces V and bending moments M in a beam are functions of the distance x measured along the
longitudinal axis. When designing a beam, it is desirable to know the values of V and M at all cross sections. A convenient
way to provide this information is to draw graphs showing how V and M vary with x. To plot such a graph, we take the
abscissa as the position of the cross section (the distance x), and we take the ordinate as the corresponding value of either
the shear force or the bending moment. These graphs are called shear force and bending moment diagrams.
The discontinuities in the shear diagram are joined by vertical lines drawn up or down to represent abrupt changes
in shear caused respectively by upward or downward concentrated loads. Discontinuities in the moment diagram are
likewise joined by straight lines to represent abrupt changes in bending moment caused by applied couples.
35

EXERCISES

Write shear and moment equations for the beams in the following problems. Draw the shear and moment diagrams,
specifying values at all change of loading positions and at all points of zero shear. Neglect the mass of the beam in each
problem.

1.

2. 60 kN
25 kN/m
36

20 kN/m
3.

4.
37

5.

6.
38

7.

8.
39

4-b.2. Relations Among Load, Shear and Moment


The relations among loads, shears, and bending moments in any beam provide a method of constructing shear and
moment diagrams without writing shear and moment equations. The relations are not independent of the basic definitions
of shear and moment; instead, they supplement them and are used in conjunction with them.
Consider the beam shown in Fig. 4-9a which is subjected to an arbitrary load w = w(x) and a series of
concentrated forces and couple moments. A free-body diagram of a small segment of the beam of length x is chosen at a
point x along the beam which is not subjected to a concentrated force or couple moment, Fig. 4-9b. The internal shear
force and bending moment shown on the free-body diagram are assumed to act in the positive sense according to the
established sign convention. The shear force and moment acting on the right-hand face of the segment is increased by a
small finite amount in order to keep it in equilibrium. The distributed loading has a resultant force F = w(x) x that acts
at a fractional distance k(x) from the right end, where 0 < k < 1.

(a) (b)
Fig. 4-9
Relation Between Load and shear
Applying the conditions of static equilibrium to Fig. 4-9b, we find that the summation of vertical forces yields
[ Fy = 0 ] V + w(x)x ( V + V ) = 0
which reduces to
V = w(x)x
Dividing by x and letting x 0, we get
dV
w ( x) Equation 4.1
dx
Thus, the slope of the shear diagram is equal to the intensity of the distributed load.
If we rewrite the above equation in the form dV w ( x) dx and perform an integration between any two points B
and C on the beam, we see that
V w( x) dx
or V = (Area)load Equation 4.2

Relation Between Shear and Moment


If we apply the moment equation of equilibrium about point O on the free-body diagram in Fig. 4-9b, we get
[ Fy = 0 ] V + w(x)x ( V + V ) = 0
[ MO = 0 ] (M + M) [w(x)x ] kx V x M = 0
M = V x + k w(x) x2
Dividing both sides of this equation by x and letting x 0, yields
dM
V Equation 4.3
dx
Therefore, the slope of the moment diagram is equal to the ordinate of the shear diagram. In particular, the absolute
dM
maximum bending moment occurs at the point where the slope 0 , or where the shear is equal to zero.
dx
If Equation 4.3 is rewritten in the form dM V dx and integrated between any two points B and C on the beam,
we have
40

M V dx
or M = (Area)shear Equation 4.4

A summary of the principles presented suggests the following procedure for the construction of shear and moment
diagrams:
1. Compute the reactions.
2. Compute the values of shear at the change of load points, using either V = (Fy )L or V = (Area)load .
3. Sketch the shear diagram, determining the shape from Equation 5.1; that is, the intensity of the load ordinate
equals the slope at the corresponding ordinate of the shear diagram.
4. Locate the points of zero shear.
5. Compute values of bending moment at the change of load points and at the points of zero shear, using either M =
(My )L = (My )R or M = (Area)shear , whichever is more convenient.
6. Sketch the moment diagram through the ordinates of the bending moments computed in step 5. The shape of the
diagram is determined from Equation 5.3; that is, the intensity of the shear ordinate equals the slope at the
corresponding ordinate of the moment diagram.

Here are important points to remember:


The slope of the shear diagram at a point is equal to the intensity of the distributed loading where positive
dV
distributed loading is upward, i.e., w(x) .
dx
If a concentrated force acts upward on the beam, the shear will jump upward by the same amount.
The change in the shear V between two points is equal to the area under the distributed-loading curve between
the points.
dM
The slope of the moment diagram at a point is equal to the shear, i.e., V .
dx
The change in the moment M between two points is equal to the area under the shear diagram between the two
points.
If a clockwise couple moment acts on the beam, the shear will not be affected; however, the moment diagram will
jump upward by the amount of the moment.
dM
Points of zero shear represent points of maximum or minimum moment since 0.
dx
Because two integrations of w = w(x) are involved to first determine the change in shear, V w( x)dx , then to


determine the change in moment, M Vdx , if the loading curve w = w(x) is a polynomial of degree n, V =
V(x) will be a curve of degree n +1, and M = M(x) will be a curve of degree n + 2.
41

EXERCISES

Draw the shear and bending moment diagrams of the beams loaded as shown.

1. 2.
42

3. 4.
43

5. 6.

`
44

7. 8.
45

9. 10.
46

12. 13. Assuming that the reaction of the ground to be


uniformly distributed, draw the shear and moment
diagrams.
47

CHAPTER 5 - ANALYSIS AND DESIGN OF BEAMS FOR BENDING

5-a. Symmetric Member in Pure Bending


Bending is a major concept in the design of many machine and structural components such as beams. Members
that are subjected to equal and opposite couples acting in the same longitudinal planes are said to be in pure bending,
where the members are assumed to possess a plane of symmetry and the couples to be acting in that plane (Fig. 5-1).
Consider a prismatic member AB possessing a plane of symmetry and subjected to equal and opposite couples M
and M' acting in that plane (Fig. 5.2a). We observe that if a section is passed through the member AB at some arbitrary
point C, the conditions of equilibrium of the portion AC of the member require that the internal forces in the section be
equivalent to the couple M (Fig. 5-2b). Thus, the internal forces in any cross section of a symmetric member in pure
bending are equivalent to a couple. The moment M of that couple is referred to as the bending moment in the section.

Fig. 5-1

(a) (b)
Fig. 5-2
Denoting by x the normal stress at a given point of the cross section and by xy and xz the components of the
shearing stress, we express that the system of the elementary internal forces exerted on the section is equivalent to the
couple M (Fig. 5-3).

Fig. 5-3
From statics, a couple consists of two equal and opposite forces. The sum of the components of these forces in
any direction is therefore equal to zero. Moreover, the moment of the couple is the same about any axis perpendicular to
its plane, and is zero about any axis contained in that plane. Selecting arbitrarily the z axis as shown in Fig. 5-3, we
express the equivalence of the elementary internal forces and of the couple M by writing that the sums of the components
and of the moments of the elementary forces are equal to the corresponding components and moments of the couple M:

x components: x dA 0
moments about y axis: z x dA 0
moments about z axis: y x dA M
The minus sign in the last equation is due to the fact that a tensile stress (x > 0) leads to a negative moment
(clockwise) of the normal force x dA about the z axis. The first equation requires that the neutral axis be a centroidal axis
while the second equation is automatically satisfied for a section that is symmetric about the y-axis.
48

5-b. Deformations in a Symmetric Member in Pure Bending


A prismatic member subjected to equal and opposite couples M and M' acting in the plane of symmetry will bend
under the action of these couples but will remain symmetric with respect to that plane (Fig. 5-4), Moreover, since the
bending moment M is the same in any cross section, the member will bend uniformly. Thus, the line AB along which the
upper face of the member intersects the plane of the couples will have a constant curvature. In other words, the line AB,
which was originally a straight line, will be transformed into a circle of center C, and so will the line A'B' along which the
lower face of the member intersects the plane of symmetry. It should also be noted that the line AB will decrease in length
when the member is bent as shown in the figure, i.e., when M > 0, while A'B' will become longer.

Fig. 5-4
The only nonzero stress component exerted on any element is the normal component x; thus, any point of a
slender member in pure bending is in a state of uniaxial stress. Recalling that, for M > 0, lines AB and A'B' are observed,
respectively, to decrease and increase in length, we note that the strain x and the stress x are negative in the upper
portion of the member (compression) and positive in the lower portion (tension). It follows that there must exist a surface
parallel to the upper and lower faces of the member, where x and x are zero. This surface is called the neutral surface.
The neutral surface intersects the plane of symmetry along an arc of circle DE (Fig. 5-5a) and it intersects a transverse
section along a straight line called the neutral axis of the section (Fig. 5-5b).

Fig. 5-5
Denoting by the radius of arc DE (Fig. 4-5a), by the central angle corresponding to DE, and observing that the
length of DE is equal to the length L of the undeformed member, we write
L =
Considering the arc JK located at a distance y above the neutral surface, we note that its length L' is
L' = ( y)
Since the original length of arc JK was equal to L, the deformation of JK is
= L' L
or, substituting,
= ( y) = y
49

The longitudinal strain x in the elements of JK is obtained by dividing by the original length L of JK.
y
x
L
y
or x Equation 5.1

The minus sign is due to the fact that we have assumed the bending moment to be positive and, thus, the beam to be
concave upward.
Because of the requirement that transverse sections remain plane, identical deformations will occur in all planes
parallel to the plane of symmetry. Thus the value of the strain given by Equation 4.1 is valid anywhere, and we conclude
that the longitudinal normal strain x varies linearly with the distance y from the neutral surface.
The strain x reaches its maximum absolute value when y itself is largest. Denoting by c the largest distance from
the neutral surface (which corresponds to either the upper or the lower surface of the member), and by m the maximum
absolute value of the strain, we have
c
m

Solving for in this equation and substituting in Equation 5.1, we have
y
x m Equation 5.2
c

5-c. Stresses and Deformations in the Elastic Range


We now consider the case when the bending moment M is such that the normal stresses in the member remain
below the yield strength y. This means that, for all practical purposes, the stresses in the member will remain below the
proportional limit and the elastic limit as well. There will be no permanent deformation, and Hookes law for uniaxial
stress applies. Assuming the material to be homogenous and denoting and denoting by E its modulus of elasticity, we have
in the longitudinal x direction
x = Ex
Multiplying both members of Equation 5.2 by E, we get
y
E x ( E m )
c
y
or x ( m ) Equation 5.3
c
where m denotes the maximum absolute value of the stress. This result shows that in the elastic range, the normal stress
varies linearly with the distance y from the neutral surface (Fig. 5-6).

Fig. 5-6
The location of the neutral surface and the magnitude of the maximum value of the stress can be found from the
relations x dA 0 and y x dA M . Substituting Equation 5.3 into the first relation yields
y m
x dA 0 = c m dA = c y dA = 0
From which it follows that
y dA 0
50

This equation shows that the first moment of the cross section about its neutral axis must be zero, which means that for a
member subjected to pure bending, and as long as the stresses remain in the elastic range, the neutral axis must pass
through the centroid of the section.

From the second relation y x dA M , specifying that the z-axis should coincide with the neutral axis of the
cross section and substituting Equation 5.3, we get

y c m dA M
y

m
y dA M
2
or
c
y dA I is the moment of
2
Since, in pure bending, the neutral axis passes through the centroid of the cross section,
inertia or second moment of area with respect to the centroidal axis perpendicular to the plane of the couple M. Solving
for m , we get
Mc
m Equation 5.4
I
and the normal stress x at any distance y from the neutral axis is
My
x Equation 5.5
I
Equations 5.4 and 5.5 are called the elastic flexure formulas and the stress x caused by the bending of the member is
often referred to as the flexure stress. The negative sign in Equation 5.5 verifies that the stress is compressive (x < 0)
above the neutral axis (y > 0) when the bending moment M is positive and tensile (x > 0) when M is negative.
I
In Equation 5.4, if , which is called the elastic section modulus denoted by S, is substituted, we get
c
M
x Equation 5.6
S
The deformation of the member caused by the bending moment M is measured by the curvature of the neutral
c
surface. The curvature is defined as the reciprocal of the radius of curvature and can be obtained from m ; i.e..

1 m

c
m
But from Hookes Law, m . Substituting m and Equation 5.4, we have
E
1 m 1 Mc

Ec Ec I
1 M
or Equation 5.7
EI
51

EXERCISES

1. Knowing that the couple shown acts in a vertical plane, determine the stress at points A and B. (Beer, 2006)
Ans. A = 61.6 MPa; B = 91.7 MPa

2. Two vertical forces are applied to a beam of the cross section shown. Determine the maximum tensile and
compressive stresses in portion BC of the beam. (Beer, 2006) Ans. c = 14.71 ksi; t = 8.82 ksi

15 kips 15 kips

3. Knowing that for the casting shown the allowable stress is 6 ksi in tension and 15 ksi in compression, determine the
largest couple M that can be applied.

4. Knowing that the allowable bending stress for the steel strip AB is 165 MPa, determine a) the largest couple M that
can be applied, and b) the corresponding radius of curvature. Use E = 200 GPa.
52

5-d. Non-uniform Bending


Consider a simply supported beam AB carrying two concentrated loads and a uniformly distributed load
(Fig. 5-7). After computing for the reactions at the supports, we pass a section through C and analyze the free-body
diagram of AC from which the shear force V and the bending couple M can be determined.

Fig. 5-7
The bending couple M creates normal stresses in the cross section, while the shear force V creates shearing
stresses in that section. Since the distribution of the normal stresses in a given section depends only upon the value of the
bending moment M in that section and the geometry of the section, the elastic flexure formulas derived in the preceding
chapter can be used to determine the maximum stress, as well as the stress at any given point, in the section. Hence,
Mc My
m x Equation 5.8
I I
where I is the moment of inertia of the crosss section with respect to a centroidal axis perpendicular to the plane of the
couple, y is the distance from the neutral surface, and c is the maximum value of that distance. In terms of the elastic
I
section modulus S = of the beam, the maximum value m of the normal stress in the section can be expressed as
c
M
x
S
The fact that m is inversely proportional to S underlines the importance of selecting beams with large section modulus.
This equation also shows that for a beam of uniform cross section, m is proportional to M . Thus, the maximum value of
the normal stress in the beam occurs in the section where M is largest. It follows that one of the most important parts of
the design of a beam for a given loading condition is the determination of the location and magnitude of the largest
bending moment.
d bd 3
For a rectangular beam of width b and depth d, c = and I , hence,
2 12
6M
max Equation 5.9
bd 2

EXERCISES:

1. For the beam and loading shown, determine the maximum normal stress due to bending on a transverse section at C.
(Beer, 2006) Ans. 10.89 MPa
53

2. For the beam and loading shown, determine the maximum normal stress due to bending on a transverse section at C.
(Beer, 2006)

3. A cantilever beam 50 mm wide by 150 mm high and 6 m long carries a load that varies uniformly from zero at the
free end to 1000 N/m at the wall. a) Compute the magnitude and location of the maximum flexural stress. b)
Determine the type and magnitude of the stress in a fiber 20 mm from the top of the beam at a section 2 m from the
free end. (Pytel, 1987) Ans. a) 32 MPa; b) 869.136 kPa
1000 N/m

150 mm

6m
50 mm

4. 50-mm diameter bar is used as a simply supported beam 3 m long. Determine the largest uniformly distributed load
that can be applied over the right two-thirds of the beam if the flexural stress is limited to 50 MPa. (Pytel, 1987)
Ans. 690.29 N/m

50 mm

5. For the beam and loading shown, determine the maximum normal stress due to bending. The section modulus for the
section is 2400 103 mm3. (Beer, 2006) Ans. 136 MPa

6. For the beam and loading shown, determine the maximum normal stress due to bending. (Beer, 2006) Ans. 30.3
MPa
54

7. For the beam and loading shown, determine the maximum normal stress due to bending. (Beer, 2006) Ans. 17.19 MPa

8. A box beam is composed of four planks, each 2 in. by 8 in., securely spiked together to form the section shown. If
wo = 300 lb/ft, find P to cause a maximum flexural stress of 1400 psi. (Pytel, 1987) Ans. 6680.656 lb

9. A wood beam 6 in. wide by 12 in. deep is loaded as shown. If the maximum flexural stress is 1200 psi, find the
maximum values of wo and P that can be applied simultaneously. (Pytel, 1987) Ans. w = 800 lb/ft; P = 7200 lb

12 in

6 in

10. Beam AB supports a uniformly distributed load of 2 kN/m and two concentrated loads P and Q. It has been
experimentally determined that the normal stress due to bending in the bottom edge of the beam is 56.9 MPa at A
and 29.9 MPa at C. Determine the magnitudes of the loads P and Q. (Beer, 2006) Ans. P = 500 N; Q = 250 N
55

5-e. Design of Prismatic Beams for Bending / Economic Sections


The design of a beam is usually controlled by the maximum absolute value of the bending moment that will occur
in the beam. The largest normal stress in the beam is found at the surface of the beam in the critical section where the
maximum moment occurs and can be obtained from Eq. 5.8, which can be written as
M max
m
S
A safe design requires that m all, where all is the allowable stress for the material used. Substituting all for m in the
above equation and solving for S yields the minimum allowable value of the section modulus for the beam being
designed:
M max
S min
all
A proper procedure in the design of beams should lead to the most economical design. This means that
among beams of the same type and the same material and other things being equal, the beam with the smallest
weight per unit length (the smallest cross-sectional area) should be selected, since this beam will be the least
expensive.

EXERCISES

1. Determine the maximum height h of the beam shown if the flexural stress is not to exceed 20 MPa. (Pytel, 1987)
Ans. 136.93 mm

2. For the beam and loading shown, design the cross section of the beam, knowing that the grade of timber used has an
allowable normal stress of 12 MPa.

3. For the beam and loading shown, design the cross section of the beam, knowing that the grade of timber used has an
allowable normal stress of 12 MPa.
56

4. Knowing that the allowable stress for the steel used is 160 MPa, select the most economical wide-flange beam to
support the loading shown.

5. Knowing that the allowable stress for the steel used is 24 ksi, select the most economical S-shape beam to
support the loading shown.

6. Two L102 76 rolled-steel angles are bolted together and used to support the loading shown. Knowing that the
allowable normal stress for the steel used is 140 MPa, determine the minimum angle thickness that can be used.

5-f. Unsymmetrical Beams


For materials that are equally strong in tension and compression, symmetrical beam sections are desirable; but for
materials that are not equally strong in tension and compression, it is desirable to use beam sections that are not
symmetrical with respect to the neutral axis. With such a section, the stronger fibers can be located at a greater distance
from the neutral axis than the weaker fibers. The ideal treatment for such materials is to locate the centroidal or neutral
axis in such a position that the ratio of the distances from it to the fibers in tension and compression is exactly the same as
the ratio of the allowable stresses in tension and in compression.

EXERCISES:

1. The inverted T section of a 4-m simply supported beam has the properties shown in the figure. The beam carries a
uniformly distributed load of intensity wo over its entire length. Determine wo if t 40 MPa and c 80 MPa.
(Pytel, 1987) Ans. 6000 N/m

wo

200 mm
NA
4m 80 mm

I = 30 106 mm4
57

2. A beam carries a concentrated load W and a total uniformly distributed load of 4W as shown in the figure. What safe
value of W can be applied if c 100 MPa and t 60 MPa? (Pytel, 1987) Ans. 9600 N

75 mm
NA

125 mm

I = 24 106 mm4

3. A T beam supports the three concentrated load shown in the figure. Determine the maximum value of P so that t 4
ksi and c 10 ksi. (Pytel, 1987) Ans. 1469.7 lb

4. Determine the maximum tensile stress and maximum compressive stress due to the load P acting on the simple beam
AB. P = 6.2 kN, L = 3.2 m, d = 1.25 m, b = 80 mm, t = 25 mm, h = 120 mm, and h1 = 90 mm. (Gere, 2009) Ans. c =
61 MPa and t = 35.4 MPa

5. Determine the largest permissible distributed load w for the beam shown, knowing that the allowable normal stress is
80 MPa in tension and 130 MPa in compression. (Beer, 2006) Ans. 150 kN/m

6. For the beam and loading shown, determine the maximum normal stress due to bending. (Beer, 2006) Ans. 86.9 MPa
58

7. A cast-iron beam 10 m long and supported as shown carries a uniformly distributed load of intensity wo. The
allowable stresses are t 20 MPa and c 80 MPa. Find the values of x and wo so that wo is a maximum.
(Pytel, 1987) Ans. wo = 3164 N/m

8. A beam ABC with an overhang from B to C supports a uniform load of 200 lb/ft throughout its length. The beam is a
channel section with dimensions as shown in the figure. The moment of inertia about the neutral axis equals 5.14 in 4.
Calculate the maximum tensile stress and the maximum compressive stress due to the uniform load. (Gere, 2001) Ans.
maximum t = 8194.55 psi; maximum c = 14568.09 psi.

9. Beams AB, BC, and CD have the cross section shown and are pin-connected at B and C. Knowing that the allowable
normal stress is 110 MPa in tension and 150 MPa in compression, determine a) the largest permissible value of w is
the beam BC is not to be overstressed, b) the corresponding maximum distance a for which the cantilever beams AB
and CD are not overstressed. (Beer, 2006) Ans. a) 1.485 kN/m; b) 1.935 m
59

5-g. Members Made of Different Materials


When members are made of several materials, the neutral axis generally does not pass through the centroid of the
composite cross section. Using the ratio of the moduli of elasticity of the materials, a transformed section is obtained
corresponding to an equivalent member made entirely of one material. To obtain the transformed section, the width of
E2
each element of the lower portion will be multiplied by the factor n, where n = . Note that this widening (if n > 1), or
E1
narrowing (if n < 1), must be effected in a direction parallel to the neutral axis of the section, since it is essential that the
distance y of each element from the neutral axis remain the same.
To obtain the stress 1 at a point in the upper portion of the cross section of the original composite bar, we simply
compute the stress x at the corresponding point of the transformed section. To obtain the stress 2 at a point in the lower
portion of the cross section, multiply by n the stress x computed at the corresponding point of the transformed section.

Fig. 5-8

PROBLEMS

1. A bar having the cross section shown has been formed by securely bonding brass and aluminum stock. Determine the
largest permissible bending moment when the composite bar is bent about a horizontal axis. For aluminum, E = 70
MPa and allow = 100 MPa; for brass, E = 105 MPa and allow = 160 MPa. Ans. 2.22 kN

2. Three wooden beams and two steel plates are securely bolted together to form the composite member shown. Using
the data given below, determine the largest permissible bending moment when the member is bent about a horizontal
axis. (Beer, 2006) Ans. 35.2 kN-m
60

3. A steel bar (Es = 210 GPa) and an aluminum bar (Ea = 70 GPa) are bonded together to form the composite bar shown.
Determine the maximum stress in the aluminum and the steel when the bar is bent about a horizontal axis, with
M = 60 N-m. (Beer, 2006) Ans. s = 81.8 MPa; a = 45.1 MPa

4. The 150250 mm timber beam has been strengthened by bolting to it the steel reinforcement shown. The modulus of
elasticity for wood is 12 GPa and for steel 200 GPa. Knowing that the beam is bent about a horizontal axis by a
couple of moment M = 50 kN-m, determine the maximum stress in wood and in steel. (Beer, 2006)
Ans. w = 18.88 MPa; s = 186.69 MPa

5-h. Eccentric Axial Loading in a Plane of symmetry


The distribution of stresses in the cross section of a member under axial loading can be assumed uniform only if
the axial loads pass through the centroid of the cross section. Such a loading is said to be centric. It follows then that the
distribution of stresses when the line of action of the loads does not pass through the centroid of the cross section, i.e.,
when the loading is eccentric, is not uniform.

(a)

(b)
Fig. 5-9 Fig. 5-10
In Fig. 5-9a is shown a member with a plane of symmetry where the axial loads are applied. The internal forces
acting on a given cross section may be represented by F applied at the centroid C of the cross section and a couple M
acting in the plane of symmetry of the member (Fig. 5-9b). The conditions of equilibrium of the free body AC require that
the force F be equal and opposite to P' and that the moment of the couple M be equal and opposite to the moment of P'
about C. Denoting by d the distance from the centroid C to the line of action AB of the forces P and P', we have
F=P and M = Pd
If the straight portion DE of member AB had been detached from AB and subjected to the centric loads P and P'
and the bending couples M and M' , the internal forces in the section would have been represented by the same force and
couple (Fig. 5-10). Thus, the stress distribution due to the original eccentric loading can be obtained by superposing the
61

uniform stress distribution corresponding to the centric loads P and P' and the linear distribution corresponding to the
bending couples M and M' (Fig. 5-11). Hence,
x ( x ) centric ( x ) bending
P My
or x Equation 5.10
A I

Fig. 5-11
EXERCISES
1. Three axial loads, each of magnitude P = 40 kN, can be applied to the end of a W20031.3 rolled-steel shape.
Determine the stress at point A, a) for the loading shown, b) if loads are applied at points 1 and 2 only. (Beer, 2006)
Ans. a) 30 MPa; b) 32 MPa

2. Knowing that the allowable stress in section ABD is 70 MPa, determine the largest force P that can be applied to the
bracket shown. (Beer, 2006) Ans. 2.62 kN

3. The eccentric axial force P acts at point D, which must be located 25 mm below the top surface of the steel bar
shown. For P = 60 kN, determine a) the depth d of the bar for which the tensile stress at point A is maximum, b) the
corresponding stress at point A. (Beer, 2006) Ans. a) 75 mm; b) 40 MPa
62

4. A short length of a rolled-steel column supports a rigid plate on which two loads P and Q are applied as shown. The
strains at two points A and B on the centerline of the outer faces of the flanges have been measured and found to be A
= 40010-6 in/in and B = 30010-6 in/in. Knowing that E = 29106 psi, determine the magnitude of each load.
(Beer, 2006) Ans. P = 44.2 kips; Q = 57.3 kips

5. The four forces shown are applied to a rigid plate supported by a solid steel post of radius a. Knowing that P = 100
kN and a = 40 mm, determine the maximum stress in the post when a) the force at D is removed, b) the forces at C
and D are removed. (Beer, 2006) Ans. a) 139.3 MPa; b) 152.3 MPa
63

CHAPPTER 6 - SHEARING STRESS IN BEAMS

6-a. Analysis of Flexure Action


Most beams are subjected to loads that produce both bending moments and shear forces (nonuniform bending). In
these cases, both normal and shear stresses are developed in the beam.
If a beam were composed of many thin layers placed on each other, bending would produce the effect shown in
Fig. 6-1. The separate layers would slide past each other and the total strength of the beam would be the sum of the
strengths of the various layers. Suppose that the layers are glued together along the contact surfaces so that they become a
single solid beam. When this beam is loaded, horizontal shear stresses must develop along the glued surfaces in order to
prevent the sliding shown in the figure. Because of the presence of these shear stresses, the single solid beam is much
stiffer and stronger than the beam with separate layers.

Fig. 6-1
Vertical and Horizontal Shear Stresses
Shear stresses acting on one side of an element are accompanied by shear stresses of equal magnitude acting on
perpendicular faces of the element. Thus, there are horizontal shear stresses acting between horizontal layers of the beam
as well as vertical shear stresses acting on the cross sections. At any point in the beam, these complementary shear
stresses are equal in magnitude.
To prove the equivalence of the horizontal and vertical shear stresses, we consider their effect on a free-body
diagram of a typical element in Fig. 6-2. A pictorial view of this element is shown in Fig. 6-3.

Fig. 6-2 Fig. 6-3

For equilibrium of this element, the shearing stress h on the bottom face requires an equal balancing shearing stress on the
top face (Fig. 6-3b). The forces causing these shearing stresses (Fig. 6-3c) form a counterclockwise couple, which requires
a clockwise couple to ensure balance. The forces of this clockwise couple induce the shearing stresses v on the vertical
faces of the element as shown. By taking moments about an axis through A, we obtain
( h dxdz)dy ( v dydz)dx 0
from which h v . We conclude therefore that a shearing stress acting on one face of an element is always
accompanied by a numerically equal shearing stress acting on a perpendicular face.
64

6-b. Shear on the Horizontal Face of a Beam Element


Consider a prismatic beam with a vertical plane of symmetry that supports various concentrated and distributed
loads. We detach from it an element CDDC of length x extending across the width of the beam from the upper surface
of the beam to a horizontal plane located at a distance y1 from the neutral axis (Fig. 6-4). The forces exerted on this
element consist of vertical shearing forces VC and VD, a horizontal shearing force H exerted on the lower face of the
element, elementary horizontal normal forces C dA and D dA, and a load wx (Fig. 6-5).

(a) (b)
Fig. 6-4 Fig. 6-5
From equilibrium,
Fx 0] H + =0

My
Replacing the flexural stress by its equivalent , we obtain
I
MD MC M D MC
H = =
I I I
The integral represents the static moment of area, which is the first moment of the cross-sectional area with respect to the
neutral axis of the portion of the cross section of the beam that is located above the line y = y1 and is denoted by Q. The
increment MD MC of the bending moment is expressed as
dM
MD MC = M = x = Vx
dx
Substituting, we obtain the following expression for the horizontal shear exerted on a beam element
VQ
H x Equation 6.1
I
The horizontal shear per unit length, denoted by q, is obtained by dividing both members of Equation 6.1 by x.
H VQ
q Equation 6.2
x I
This horizontal shear per unit length is called shear flow.

6-b.1. Longitudinal Shear on a Beam Element of Arbitrary Shape


The equation derived in the preceding section can be used to find the shear on a longitudinal surface but Q will be
equal to the first moment of the shaded area only with respect to the neutral axis.

Fig. 6-6
65

6-b.2. Horizontal Shearing Stress Formula


Consider the same beam as in the preceding section. The magnitude H of the shearing force exerted on the
horizontal face of the element can be obtained from Equation 6.2. The average shearing stress ave on that face of the
element is obtained by dividing H by the area A of the face, which is equal to bx, where b is the width of the element
at the cut. Hence,
H VQ x
ave =
x I bx
b VQ
ave Equation 6.3
Ib

Fig. 6-7

Since the shearing stresses exerted respectively on a transverse and a horizontal plane through plane D are equal
( h v ), the expression obtained also represents the average value of shear along the line D1 and D2.

6-b.3. Application to Rectangular Section


The distribution of shear stress in a rectangular section can be obtained by applying Equation 6.3 to Fig. 6-8. For a
layer at a distance y from the neutral axis, we have

V h
A' y b y y y
V 1 h
=
Ib Ib 2 22
which reduces to

V h2 2
y
2I 4
Fig. 6-8
This shows that the shearing stress is distributed parabolically across the depth of the section. The maximum shearing
stress occurs at the neutral axis and is found by substituting the dimensions of the rectangle in Equation 6.3 as follows:
A' y 3
V V bh h
=
Ib bh 2 4
b
12
which reduces to
3V
Max. =
2 bh
3V
Max. = Equation 6.4
2A
This indicates that the maximum shearing stress in a rectangular section is 50% greater than the average shear stress.
66

EXERCISES:

1. Three boards, each of 3890 mm rectangular cross section, are nailed together to form a beam that is subjected to a
vertical shear of 1 kN. Knowing that the spacing between each pair of nails is 60 mm, determine the shearing force in
each nail. Ans. 351 N

2. The American Standard rolled-steel beam shown has been reinforced by attaching to it two 16200 mm plates, using
18-mm-diameter bolts spaced longitudinally every 120 mm. Knowing that the average allowable shearing stress in the
bolts is 90 MPa, determine the largest permissible vertical shearing force. For S31052, A = 6650 mm2, h = 305 mm,
Ix = 95.3106 mm4. Ans. 193.5 kN

3. The built-up timber beam is subjected to a vertical shear of 5 kN. Knowing that the allowable shearing force in the
nails is 300 N, determine the largest permissible spacing s of the nails. Ans. 40.1 mm

4. Four 1021029.5 steel angle shapes and a 12400 mm steel plate are bolted together to form a beam with cross
section shown. The bolts are of 22-mm diameter and are spaced longitudinally every 120 mm. Knowing that the beam
is subjected to a vertical shear of 240 kN, determine the average shearing stress in each bolt. For the angle section, A
= 1850 mm2, Ix = 1.83106 mm4, y = 29 mm. Ans. 83.3 MPa
67

5. A timber beam 80 mm wide by 160 mm high is subjected to a vertical shear V = 40 kN. Determine the shearing stress
developed at layers 20 mm apart from top to bottom of the section.

160 mm

80 mm

6. Two wood beams, each of square cross section (3.5 in. 3.5 in., actual dimensions) are glued together to form a solid
beam of dimensions 3.5 in. 7.0 in. The beam is simply supported with a span of 6 ft. What is the maximum load
Pmax that may act at the midpoint if the allowable shear stress in the glued joint is 200 psi? (Include the effects of the
beams own weight, assuming that the wood weighs 0.02 lb/in3). Ans. 6948 lb

7. A uniformly distributed load of 200 lb/ft is carried on a simply supported span. If the cross section is as shown in the
figure, determine the maximum length of the beam if the shearing stress is limited to 80 psi. Assume that the load acts
over the entire length of the beam. Ans. 12.636 ft

10 in.
8 in.

6 in.

8 in.

8. The T-section shown is the cross-section of a beam formed by joining two rectangular pieces of wood together. The
beam is subjected to a maximum shearing force of 60 kN. Determine the shearing stress a) at the neutral axis, and b)
at the junction between the two pieces of wood. Ans. a) 3.28 MPa; b) 3.178 MPa, 31.78 MPa
200 mm

40 mm

100 mm

20 mm
68

9. Determine the maximum and minimum shearing stress in the web of the wide-flange section in the figure if V = 100
kN. Also, compute the percentage of vertical shear carried only by the web of the beam. Ans. 90.23 %

10. A plywood beam is built up of -in. strips separated by blocks as shown in the figure. What shearing force V will
cause a maximum shearing stress of 200 psi? Ans. 1481.692 lb

6-c. Design for Flexure and Shear

1. A laminated wood beam on simple supports is built up by gluing together three 2 in. 4 in. boards (actual
dimensions) to form a solid beam 4 in. 6 in. in cross-section as shown in the figure. The allowable shear stress in the
glued joints is 50 psi and the allowable bending stress in the wood is 1500 psi. If the beam is 6 ft long, what is the
allowable load P acting at the midpoint of the beam? (Disregard the weight of the beam.) Ans. 1800 lb

2. A wood beam ABC with simple supports at A and B and an overhang BC has height h = 280 mm. The length of the
main span of the beam is L = 3.6 m and the length of the overhang is L/3 = 1.2 m. The beam supports a concentrated
load 3P = 15 kN at the midpoint of the main span and a load P = 5 kN at the free end of the overhang. The wood has
weight density = 5.5 kN/m3. a) Determine the required width b of the beam based upon an allowable bending stress
of 8.2 MPa. b) Determine the required width b of the beam based upon an allowable shear stress of 0.7 MPa.
69

3. A rectangular beam 6 in. wide by 10 in. high supports a total distributed load of W and a concentrated load of 2W
applied as shown. If f 1500 psi and = 120 psi, determine the maximum value of W. Ans. 2222 lb

10 in

6 in

4. The distributed load shown is supported by a box beam having the cross-section shown. Determine the maximum
value of wo that will not exceed a flexural stress of 10 MPa or a shearing stress of 1.0 MPa.

150 250

200

300

Dimensions are in mm

5. The beam supports two concentrated loads P and a triangular load of magnitude 3P. The beam is made of three 1/4
in thick plywood strips separated by wood blocks as shown. Determine the maximum allowable value of P if the
working stresses are 1200 psi in bending and 200 psi in shear. Ans. 740.845 lb

6. The distributed load shown is supported by a wide-flange section of the given dimensions. Determine the maximum
value of wo that will not exceed a flexural stress of 10 MPa or a shearing stress of 1.0 MPa.
70

7. The overhanging beam carries two concentrated loads W and a uniformly distributed load of magnitude 4W. The
working stresses are 6,000 psi in tension, 10,000 psi in compression, and 8,000 psi in shear. a) Show that the neutral
axis of the cross section is located at d = 2.167 in. and that the moment of inertia of the cross sectional area about the
neutral is I = 61.55 in4. b) Determine the largest allowable value of W. Ans. 13370.728 lb

8. The wood beam has a square section. Find the smallest allowable cross sectional dimensions if the working stresses
are 8 MPa in bending and 1.0 MPa in shear. Ans. 144.23 mm

9. A box beam carries a distributed load of 200 lb/ft and a concentrated load of P as shown in the figure. Determine the
maximum value of P if f 1200 psi and = 150 psi.

You might also like